Funciones de onda no integrables

Supongamos que la perturbación de primer orden produce una corrección creíble de la energía, pero una corrección de la función de onda que no es integrable al cuadrado. Eso puede suceder, no veo ninguna razón por la que no podría. A menos que haya alguna prueba de que no puede suceder (no he encontrado ninguna). Y si sucede, ¿ creerías en el cambio de energía que calculaste?

En mi opinión, eso debería estar prohibido por el teorema de Kato-Rellich. Pero voy a comprobar sólo para estar seguro.
Sí, gracias.... El punto es que, en la teoría de la perturbación, la corrección de primer orden de la función propia se expresa como una combinación lineal de todas las funciones propias no perturbadas. Cada una de las funciones propias no perturbadas es integrable al cuadrado, por supuesto, pero la serie puede producir una función no integrable.
Las correcciones en sí dependen de su potencial. La pregunta entonces es qué potencial permite que se agregue a su problema. Tomando como ejemplo el átomo de hidrógeno, obtenemos una corrección que se comporta como Σ metro 2 norte 2 metro 2 norte 2 < norte ( 0 ) | V | metro ( 0 ) > . Si podemos introducir un potencial cuyos elementos de la matriz decaigan más lentamente que norte 2 metro 2 , podemos obtener una serie divergente.
Ese es un problema delicado, debe ser discutido en el Capítulo VI de Kato, Teoría de Perturbación para Operadores Lineales, Springer 1966
Tengo curiosidad por saber el contexto en el que surge esto.
No es exactamente lo que estás preguntando, pero considera el oscilador armónico hamiltoniano. H con un λ X 4 perturbación. Esto tiene correcciones "creíbles" para los valores propios y los vectores propios, pero para λ < 0 , H + λ X 4 no tiene valores propios discretos, por lo que no puede creer los cálculos formales de perturbación.
me recuerda a la pregunta similar pero más amplia de 0celo7 sobre si "el descuido matemático en QM alguna vez produce predicciones incorrectas" physics.stackexchange.com/questions/348913/…

Respuestas (2)

Creo que la prueba de wiki para el término de corrección de primer orden es bastante clara. https://en.wikipedia.org/wiki/Perturbation_theory_(quantum_mechanics)#First_order_corrections

La oración importante que se conecta a su pregunta es "Let V sea ​​un hamiltoniano que represente una perturbación física débil". El núcleo de la pertubación es la linealización bajo pequeños cambios, así que no piense V como un potencial habitual que contiene singularidades, si lo hace, probablemente signifique que su sistema está confinado en una región mucho más pequeña por una pared infinita.

Entonces, antes de obtener el término de corrección, existe la expresión ( mi norte ( 0 ) mi k ( 0 ) ) k ( 0 ) | norte ( 1 ) = k ( 0 ) | V | norte ( 1 ) lo que garantizó que el RHS es un número finito pequeño, y cada vector de base turbado (al menos el primero) es integrable al cuadrado (integrable al cuadrado significa que puede obtener un número finito, no la integabilidad de Riemann).

Si construiste un V de modo que no es integrable al cuadrado, entonces no debería usar la teoría de la pertubación, porque eso es una "patada", no una pertubación.

Desde los conceptos básicos de QM , las únicas funciones de onda que son integrables al cuadrado son físicas. Por lo tanto, las funciones de onda que no son integrables al cuadrado solo pueden aparecer en matemáticas, pero ningún sistema físico puede tener tal función de onda .

Además del hamiltoniano original , las correcciones a energías y funciones de onda sólo dependen de las perturbaciones ( interacción hamiltoniana ). Al restringir las funciones de onda físicamente aceptables , podemos restringir esencialmente las perturbaciones permitidas en el sistema original dado.

Lo bueno es que este tipo de restricciones en las interacciones permitidas ocurre naturalmente en el contexto de la Teoría Cuántica de Campos (QFT) . Invocando la invariancia de Gauge en el lagrangiano original .

Por ejemplo, al invocar la invariancia de Gauge en el lagrangiano libre de Dirac obtenemos la electrodinámica cuántica , QED , Lagrangiano que solo tiene un término de interacción correspondiente a la interacción fermión-fotón.

Además, en QFT las interacciones no físicas se pueden descartar mediante análisis dimensional y renormalizabilidad de la teoría.

Entonces, ¿no hay partículas libres para ti?
@lalala a que te refieres?
Las funciones propias del hamiltoniano de partículas libres no son integrables al cuadrado.
@lalala No. Es cuadrado integrable. Para una partícula libre: \psi_k(r) = [e^{ik.r}]/\sqrt{V} y \integral dr |\psi_k(r)|^2 = 1.
No es integrable ya que ψ ψ es un constante, por lo que la integración en todo el espacio dará infinitos ... a menos, por supuesto, V también es infinito, pero entonces tu ψ k ( r ) es cero en todas partes...